Proof involving the difference of iid normal random variables












0












$begingroup$


I am trying to prove that, for two i.i.d. random variables $X_1,X_2 sim N(mu, sigma^2)$, their difference divided by $sqrt 2$ is standard normal, i.e. $frac {X_1-X_2}{sqrt 2} sim N(0,1)$.



I am trying to show it using convolution, but am getting caught up in the algebra of trying to get it into a recognizable form. I think there is a more simple way using the definition of expectation, but when I try it, it doesn't quite come out right as I get $N(0, sigma^2)$ instead of $N(0,1)$.










share|cite|improve this question











$endgroup$












  • $begingroup$
    @angryavian So the problem I am having with the linear combination approach comes down to the variance. I am trying to calculate as follows: Var[(X_1-X_2)/sqrt(2)]=(1/2)Var[X_1-X_2]=2(1/2)Var[X_1]=sigma^2 So I'm getting sigma^2 instead of 1. Can you help me find my mistake?
    $endgroup$
    – eyehearyou
    Jan 29 at 5:13








  • 1




    $begingroup$
    It's not a mistake. You need to divide by $sqrt{2}sigma$ to get $N(0,1)$.
    $endgroup$
    – d.k.o.
    Jan 29 at 5:20










  • $begingroup$
    The simplest way to prove this is to use characteristic functions. Are you allowed to do that?. (As already pointed out you will not get $N(0,1)$ if you divide by $sqrt 2$).
    $endgroup$
    – Kavi Rama Murthy
    Jan 29 at 5:31












  • $begingroup$
    @KaviRamaMurthy I'm allowed to do anything. This is not for a homework or school assignment, I'm just reading my way through a textbook and came upon an something that I couldn't immediately see how to prove. Thanks!
    $endgroup$
    – eyehearyou
    Jan 29 at 5:34


















0












$begingroup$


I am trying to prove that, for two i.i.d. random variables $X_1,X_2 sim N(mu, sigma^2)$, their difference divided by $sqrt 2$ is standard normal, i.e. $frac {X_1-X_2}{sqrt 2} sim N(0,1)$.



I am trying to show it using convolution, but am getting caught up in the algebra of trying to get it into a recognizable form. I think there is a more simple way using the definition of expectation, but when I try it, it doesn't quite come out right as I get $N(0, sigma^2)$ instead of $N(0,1)$.










share|cite|improve this question











$endgroup$












  • $begingroup$
    @angryavian So the problem I am having with the linear combination approach comes down to the variance. I am trying to calculate as follows: Var[(X_1-X_2)/sqrt(2)]=(1/2)Var[X_1-X_2]=2(1/2)Var[X_1]=sigma^2 So I'm getting sigma^2 instead of 1. Can you help me find my mistake?
    $endgroup$
    – eyehearyou
    Jan 29 at 5:13








  • 1




    $begingroup$
    It's not a mistake. You need to divide by $sqrt{2}sigma$ to get $N(0,1)$.
    $endgroup$
    – d.k.o.
    Jan 29 at 5:20










  • $begingroup$
    The simplest way to prove this is to use characteristic functions. Are you allowed to do that?. (As already pointed out you will not get $N(0,1)$ if you divide by $sqrt 2$).
    $endgroup$
    – Kavi Rama Murthy
    Jan 29 at 5:31












  • $begingroup$
    @KaviRamaMurthy I'm allowed to do anything. This is not for a homework or school assignment, I'm just reading my way through a textbook and came upon an something that I couldn't immediately see how to prove. Thanks!
    $endgroup$
    – eyehearyou
    Jan 29 at 5:34
















0












0








0





$begingroup$


I am trying to prove that, for two i.i.d. random variables $X_1,X_2 sim N(mu, sigma^2)$, their difference divided by $sqrt 2$ is standard normal, i.e. $frac {X_1-X_2}{sqrt 2} sim N(0,1)$.



I am trying to show it using convolution, but am getting caught up in the algebra of trying to get it into a recognizable form. I think there is a more simple way using the definition of expectation, but when I try it, it doesn't quite come out right as I get $N(0, sigma^2)$ instead of $N(0,1)$.










share|cite|improve this question











$endgroup$




I am trying to prove that, for two i.i.d. random variables $X_1,X_2 sim N(mu, sigma^2)$, their difference divided by $sqrt 2$ is standard normal, i.e. $frac {X_1-X_2}{sqrt 2} sim N(0,1)$.



I am trying to show it using convolution, but am getting caught up in the algebra of trying to get it into a recognizable form. I think there is a more simple way using the definition of expectation, but when I try it, it doesn't quite come out right as I get $N(0, sigma^2)$ instead of $N(0,1)$.







probability statistics






share|cite|improve this question















share|cite|improve this question













share|cite|improve this question




share|cite|improve this question








edited Jan 29 at 5:09









Mohammad Zuhair Khan

1,6792625




1,6792625










asked Jan 29 at 4:55









eyehearyoueyehearyou

306




306












  • $begingroup$
    @angryavian So the problem I am having with the linear combination approach comes down to the variance. I am trying to calculate as follows: Var[(X_1-X_2)/sqrt(2)]=(1/2)Var[X_1-X_2]=2(1/2)Var[X_1]=sigma^2 So I'm getting sigma^2 instead of 1. Can you help me find my mistake?
    $endgroup$
    – eyehearyou
    Jan 29 at 5:13








  • 1




    $begingroup$
    It's not a mistake. You need to divide by $sqrt{2}sigma$ to get $N(0,1)$.
    $endgroup$
    – d.k.o.
    Jan 29 at 5:20










  • $begingroup$
    The simplest way to prove this is to use characteristic functions. Are you allowed to do that?. (As already pointed out you will not get $N(0,1)$ if you divide by $sqrt 2$).
    $endgroup$
    – Kavi Rama Murthy
    Jan 29 at 5:31












  • $begingroup$
    @KaviRamaMurthy I'm allowed to do anything. This is not for a homework or school assignment, I'm just reading my way through a textbook and came upon an something that I couldn't immediately see how to prove. Thanks!
    $endgroup$
    – eyehearyou
    Jan 29 at 5:34




















  • $begingroup$
    @angryavian So the problem I am having with the linear combination approach comes down to the variance. I am trying to calculate as follows: Var[(X_1-X_2)/sqrt(2)]=(1/2)Var[X_1-X_2]=2(1/2)Var[X_1]=sigma^2 So I'm getting sigma^2 instead of 1. Can you help me find my mistake?
    $endgroup$
    – eyehearyou
    Jan 29 at 5:13








  • 1




    $begingroup$
    It's not a mistake. You need to divide by $sqrt{2}sigma$ to get $N(0,1)$.
    $endgroup$
    – d.k.o.
    Jan 29 at 5:20










  • $begingroup$
    The simplest way to prove this is to use characteristic functions. Are you allowed to do that?. (As already pointed out you will not get $N(0,1)$ if you divide by $sqrt 2$).
    $endgroup$
    – Kavi Rama Murthy
    Jan 29 at 5:31












  • $begingroup$
    @KaviRamaMurthy I'm allowed to do anything. This is not for a homework or school assignment, I'm just reading my way through a textbook and came upon an something that I couldn't immediately see how to prove. Thanks!
    $endgroup$
    – eyehearyou
    Jan 29 at 5:34


















$begingroup$
@angryavian So the problem I am having with the linear combination approach comes down to the variance. I am trying to calculate as follows: Var[(X_1-X_2)/sqrt(2)]=(1/2)Var[X_1-X_2]=2(1/2)Var[X_1]=sigma^2 So I'm getting sigma^2 instead of 1. Can you help me find my mistake?
$endgroup$
– eyehearyou
Jan 29 at 5:13






$begingroup$
@angryavian So the problem I am having with the linear combination approach comes down to the variance. I am trying to calculate as follows: Var[(X_1-X_2)/sqrt(2)]=(1/2)Var[X_1-X_2]=2(1/2)Var[X_1]=sigma^2 So I'm getting sigma^2 instead of 1. Can you help me find my mistake?
$endgroup$
– eyehearyou
Jan 29 at 5:13






1




1




$begingroup$
It's not a mistake. You need to divide by $sqrt{2}sigma$ to get $N(0,1)$.
$endgroup$
– d.k.o.
Jan 29 at 5:20




$begingroup$
It's not a mistake. You need to divide by $sqrt{2}sigma$ to get $N(0,1)$.
$endgroup$
– d.k.o.
Jan 29 at 5:20












$begingroup$
The simplest way to prove this is to use characteristic functions. Are you allowed to do that?. (As already pointed out you will not get $N(0,1)$ if you divide by $sqrt 2$).
$endgroup$
– Kavi Rama Murthy
Jan 29 at 5:31






$begingroup$
The simplest way to prove this is to use characteristic functions. Are you allowed to do that?. (As already pointed out you will not get $N(0,1)$ if you divide by $sqrt 2$).
$endgroup$
– Kavi Rama Murthy
Jan 29 at 5:31














$begingroup$
@KaviRamaMurthy I'm allowed to do anything. This is not for a homework or school assignment, I'm just reading my way through a textbook and came upon an something that I couldn't immediately see how to prove. Thanks!
$endgroup$
– eyehearyou
Jan 29 at 5:34






$begingroup$
@KaviRamaMurthy I'm allowed to do anything. This is not for a homework or school assignment, I'm just reading my way through a textbook and came upon an something that I couldn't immediately see how to prove. Thanks!
$endgroup$
– eyehearyou
Jan 29 at 5:34












1 Answer
1






active

oldest

votes


















0












$begingroup$

$Ee^{itX_1}=e^{itmu}e^{-t^{2}sigma^{2}/2}$. Hence $Ee^{it(X_1-X_2)/sqrt2 sigma)}=Ee^{itX_1/sqrt2 sigma)}Ee^{-itX_2/sqrt2 sigma)}=(e^{-t^{2}/4})^{2}=e^{-t^{2}/2}$ which is the characteristic function of $N(0,1)$.






share|cite|improve this answer









$endgroup$













  • $begingroup$
    Thanks so much Kavi! I see how this works when we have that factor of $sigma$ in the denominator. Unfortunately, the book I have just has $frac{X_1-X_2}{sqrt{2}}$ :(. Maybe it's an error, but there is an important proof that seems to rest on it, so that is rather discouraging.
    $endgroup$
    – eyehearyou
    Jan 29 at 5:45














Your Answer





StackExchange.ifUsing("editor", function () {
return StackExchange.using("mathjaxEditing", function () {
StackExchange.MarkdownEditor.creationCallbacks.add(function (editor, postfix) {
StackExchange.mathjaxEditing.prepareWmdForMathJax(editor, postfix, [["$", "$"], ["\\(","\\)"]]);
});
});
}, "mathjax-editing");

StackExchange.ready(function() {
var channelOptions = {
tags: "".split(" "),
id: "69"
};
initTagRenderer("".split(" "), "".split(" "), channelOptions);

StackExchange.using("externalEditor", function() {
// Have to fire editor after snippets, if snippets enabled
if (StackExchange.settings.snippets.snippetsEnabled) {
StackExchange.using("snippets", function() {
createEditor();
});
}
else {
createEditor();
}
});

function createEditor() {
StackExchange.prepareEditor({
heartbeatType: 'answer',
autoActivateHeartbeat: false,
convertImagesToLinks: true,
noModals: true,
showLowRepImageUploadWarning: true,
reputationToPostImages: 10,
bindNavPrevention: true,
postfix: "",
imageUploader: {
brandingHtml: "Powered by u003ca class="icon-imgur-white" href="https://imgur.com/"u003eu003c/au003e",
contentPolicyHtml: "User contributions licensed under u003ca href="https://creativecommons.org/licenses/by-sa/3.0/"u003ecc by-sa 3.0 with attribution requiredu003c/au003e u003ca href="https://stackoverflow.com/legal/content-policy"u003e(content policy)u003c/au003e",
allowUrls: true
},
noCode: true, onDemand: true,
discardSelector: ".discard-answer"
,immediatelyShowMarkdownHelp:true
});


}
});














draft saved

draft discarded


















StackExchange.ready(
function () {
StackExchange.openid.initPostLogin('.new-post-login', 'https%3a%2f%2fmath.stackexchange.com%2fquestions%2f3091766%2fproof-involving-the-difference-of-iid-normal-random-variables%23new-answer', 'question_page');
}
);

Post as a guest















Required, but never shown

























1 Answer
1






active

oldest

votes








1 Answer
1






active

oldest

votes









active

oldest

votes






active

oldest

votes









0












$begingroup$

$Ee^{itX_1}=e^{itmu}e^{-t^{2}sigma^{2}/2}$. Hence $Ee^{it(X_1-X_2)/sqrt2 sigma)}=Ee^{itX_1/sqrt2 sigma)}Ee^{-itX_2/sqrt2 sigma)}=(e^{-t^{2}/4})^{2}=e^{-t^{2}/2}$ which is the characteristic function of $N(0,1)$.






share|cite|improve this answer









$endgroup$













  • $begingroup$
    Thanks so much Kavi! I see how this works when we have that factor of $sigma$ in the denominator. Unfortunately, the book I have just has $frac{X_1-X_2}{sqrt{2}}$ :(. Maybe it's an error, but there is an important proof that seems to rest on it, so that is rather discouraging.
    $endgroup$
    – eyehearyou
    Jan 29 at 5:45


















0












$begingroup$

$Ee^{itX_1}=e^{itmu}e^{-t^{2}sigma^{2}/2}$. Hence $Ee^{it(X_1-X_2)/sqrt2 sigma)}=Ee^{itX_1/sqrt2 sigma)}Ee^{-itX_2/sqrt2 sigma)}=(e^{-t^{2}/4})^{2}=e^{-t^{2}/2}$ which is the characteristic function of $N(0,1)$.






share|cite|improve this answer









$endgroup$













  • $begingroup$
    Thanks so much Kavi! I see how this works when we have that factor of $sigma$ in the denominator. Unfortunately, the book I have just has $frac{X_1-X_2}{sqrt{2}}$ :(. Maybe it's an error, but there is an important proof that seems to rest on it, so that is rather discouraging.
    $endgroup$
    – eyehearyou
    Jan 29 at 5:45
















0












0








0





$begingroup$

$Ee^{itX_1}=e^{itmu}e^{-t^{2}sigma^{2}/2}$. Hence $Ee^{it(X_1-X_2)/sqrt2 sigma)}=Ee^{itX_1/sqrt2 sigma)}Ee^{-itX_2/sqrt2 sigma)}=(e^{-t^{2}/4})^{2}=e^{-t^{2}/2}$ which is the characteristic function of $N(0,1)$.






share|cite|improve this answer









$endgroup$



$Ee^{itX_1}=e^{itmu}e^{-t^{2}sigma^{2}/2}$. Hence $Ee^{it(X_1-X_2)/sqrt2 sigma)}=Ee^{itX_1/sqrt2 sigma)}Ee^{-itX_2/sqrt2 sigma)}=(e^{-t^{2}/4})^{2}=e^{-t^{2}/2}$ which is the characteristic function of $N(0,1)$.







share|cite|improve this answer












share|cite|improve this answer



share|cite|improve this answer










answered Jan 29 at 5:42









Kavi Rama MurthyKavi Rama Murthy

71.2k53170




71.2k53170












  • $begingroup$
    Thanks so much Kavi! I see how this works when we have that factor of $sigma$ in the denominator. Unfortunately, the book I have just has $frac{X_1-X_2}{sqrt{2}}$ :(. Maybe it's an error, but there is an important proof that seems to rest on it, so that is rather discouraging.
    $endgroup$
    – eyehearyou
    Jan 29 at 5:45




















  • $begingroup$
    Thanks so much Kavi! I see how this works when we have that factor of $sigma$ in the denominator. Unfortunately, the book I have just has $frac{X_1-X_2}{sqrt{2}}$ :(. Maybe it's an error, but there is an important proof that seems to rest on it, so that is rather discouraging.
    $endgroup$
    – eyehearyou
    Jan 29 at 5:45


















$begingroup$
Thanks so much Kavi! I see how this works when we have that factor of $sigma$ in the denominator. Unfortunately, the book I have just has $frac{X_1-X_2}{sqrt{2}}$ :(. Maybe it's an error, but there is an important proof that seems to rest on it, so that is rather discouraging.
$endgroup$
– eyehearyou
Jan 29 at 5:45






$begingroup$
Thanks so much Kavi! I see how this works when we have that factor of $sigma$ in the denominator. Unfortunately, the book I have just has $frac{X_1-X_2}{sqrt{2}}$ :(. Maybe it's an error, but there is an important proof that seems to rest on it, so that is rather discouraging.
$endgroup$
– eyehearyou
Jan 29 at 5:45




















draft saved

draft discarded




















































Thanks for contributing an answer to Mathematics Stack Exchange!


  • Please be sure to answer the question. Provide details and share your research!

But avoid



  • Asking for help, clarification, or responding to other answers.

  • Making statements based on opinion; back them up with references or personal experience.


Use MathJax to format equations. MathJax reference.


To learn more, see our tips on writing great answers.




draft saved


draft discarded














StackExchange.ready(
function () {
StackExchange.openid.initPostLogin('.new-post-login', 'https%3a%2f%2fmath.stackexchange.com%2fquestions%2f3091766%2fproof-involving-the-difference-of-iid-normal-random-variables%23new-answer', 'question_page');
}
);

Post as a guest















Required, but never shown





















































Required, but never shown














Required, but never shown












Required, but never shown







Required, but never shown

































Required, but never shown














Required, but never shown












Required, but never shown







Required, but never shown







Popular posts from this blog

Can a sorcerer learn a 5th-level spell early by creating spell slots using the Font of Magic feature?

Does disintegrating a polymorphed enemy still kill it after the 2018 errata?

A Topological Invariant for $pi_3(U(n))$